A small problem for legendary JamesR on Relativity

Discussion in 'Physics & Math' started by RawThinkTank, Sep 27, 2004.

Thread Status:
Not open for further replies.
  1. MacM Registered Senior Member

    Messages:
    10,104
    Paul T,

    Your post merits no response. It is outright BS. Your ignorance is not my responsibility. Your opinion is clearly at the bottom of the barrel.
     
  2. Google AdSense Guest Advertisement



    to hide all adverts.
  3. Persol I am the great and mighty Zo. Registered Senior Member

    Messages:
    5,946
    You've got to be kidding me...
     
  4. Google AdSense Guest Advertisement



    to hide all adverts.
  5. MacM Registered Senior Member

    Messages:
    10,104
    Yes. Thanks for reminding me. Your opinion is at the bottom of the barrel.

    Please Register or Log in to view the hidden image!

     
  6. Google AdSense Guest Advertisement



    to hide all adverts.
  7. Persol I am the great and mighty Zo. Registered Senior Member

    Messages:
    5,946
    Good.. now that we have that straight...
     
  8. Paul T Registered Senior Member

    Messages:
    460
    Hehehe, sure...mister dumb perfect English writer. You are free to put my opinion at the bottom of the barrel [deleted] or whatever...who cares. We know the truth, though.

    [deleted]

    Please Register or Log in to view the hidden image!

     
    Last edited by a moderator: Oct 24, 2004
  9. MacM Registered Senior Member

    Messages:
    10,104
    You really shouldn't be so offended about your bad english. I'm sure I speak far less of your language, whatever it may be.

    But you really should take into consideration that at least 50% of your comments are off target as a result of your lack of understanding of what has been said.
     
  10. James R Just this guy, you know? Staff Member

    Messages:
    39,426
    I have removed some off topic material from this thread, edited out some personal insults and deleted a number of posts.

    Please conduct your personal flame wars by PM, or email, or off-forum.

    Thankyou.
     
  11. Quantum Quack Life's a tease... Valued Senior Member

    Messages:
    23,328
    let's have a little look at this statement given by Paul T.

    you clearly state that
    Now Paul why did you not clearly state that this 'kinetic energy change" was due to Petes efforts and just simply state kinetic energy change?

    When I read this I thought that this was Petes effort but was unsure that was what you intended.

    To say that it is a change in petes kinetic energy in this scenario is insufficient I would think. With out clarifying the nature of the change or where it came from.
    Can I ask whether this is a common way of expressing or was there some other motive?

    I am not surprised that in the hostile climate currently demonstrated that MacM missed this easy assumption and declared a concern about the unaccounted for 2500j or should I say "petes kinetic energy change".
     
  12. MacM Registered Senior Member

    Messages:
    10,104
    Thank you James R.
     
  13. Paul T Registered Senior Member

    Messages:
    460
    Non-relativistic solution done. Now, just for another fun, let's move on to relativistic case. We now shall use a spaceship with some sort of accelerating tube that could accelerate a load with mass (m) to velocity of 0.01c.

    Use the following figures:

    Spaceship mass, M = 1,000,000 kg (rest mass relative to an earth observer)
    Spaceship velocity (v) relative the earth observer = 0.6c
    Mass of load to be ejected = 50 kg
    Velocity (v1) of ejected load relative to the spaceship = 0.01c

    Gamma (for v=0.6c) is 1.25

    Initial momentum of "spaceship+load" = 1.25*(1,000,000+50)*0.6c = 750,037.5 kg m/s

    Use momentum conservation to get the final velocity of spaceship after ejecting the load. Also use p' = g(p+(b/c)E) = g(p+mv). Other way, similar to treatment for non-relativistic solution can be also employed.

    1,000,000u/(sqrt(1-u<sup>2</sup>/c<sup>2</sup>)) + p' = 750,037.5

    p' = 1.25*(50*0.01c+50*0.6c)=38.125 kg m/s

    Hence, u = 0.59999968c. The load velocity relative to earth observer is (0.6c+0.01c)/(1+0.6*0.01) = 0.606361829c

    Please note that there are two assumptions have been made in this computation, they are:

    1) Assume no relativistic effect for the 50 kg load in the spaceship reference frame
    2) To compute p', the spaceship slowing down movement is ignored.

    Those two assumptions were made to simplified the calculation only.

    Now, let's compute the kinetic energy change.

    a) Spaceship
    DKE = (1/(sqrt(1-0.59999968<sup>2</sup>)) - 1.25)*1,000,000c<sup>2</sup> = -0.375c<sup>2</sup>​
    b) Load
    DKE = (1/(sqrt(1-0.606361829<sup>2</sup>)) - 1.25)*50c<sup>2</sup> = -0.378c<sup>2</sup>​
    Of course, similarly as for non-relativistic solution, there is a "missing energy" here. This "missing energy" is the kinetic energy change attributed to the acceleration process. I don't know how clear this statement to some people. The balance is 1.25*(0.5*50*(0.01c)<sup>2</sup>) = 0.003125c<sup>2</sup>. So, 0.375c<sup>2</sup>+0.003125c<sup>2</sup> = 0.378125c<sup>2</sup>.

    There is a shortcut for all this, use E' = g(E+pv), where E is total energy (not kinetic energy). However, I will leave this to those who have interest on the matter.

    Sorry, I cannot make this computation any clearer for people like MacM to grasp.
     
  14. Paul T Registered Senior Member

    Messages:
    460
    Congrat, you had some common sense to know what that "missing energy" attributed to.
     
  15. MacM Registered Senior Member

    Messages:
    10,104
    To all of this I would have to simply say "So What?"

    I have never said Relativity was mathematically flawed. I have said and continue to say it is philosophically wrong or is based on false interpretations of observational information and test data.
     
  16. Paul T Registered Senior Member

    Messages:
    460
    Your opinion worth nothing in this particular case. Firstly, I was just doing the computation for fun. Secondly, I just wanted to make a relativistic solution equivalent to the one that I had made earlier for non-relativistic case. Both solutions tell us the same thing that the huge amount of kinetic energy increase according to the outside observer (earth) is not come from nowhere or that the energy is just, in that sense, an illusion of motion. It is real energy, gained by one body and lost by the other.

    The computation passed over your shoulder again, as usual. However, one thing you missed as always, the computation show nothing about ratio of 13-billions to 1 (if I am not mistaken was the figure raised by you in one of the scenario discussed under this thread a few days ago) as peculiar or that some mystical energy has to be considered to fill the huge energy shortage (13-billions times of the energy in the local frame!). The computation showed that the "energy shortage" (not really a correct term, in fact, as there is no such shortage) is supplied by the spaceship (for relativistic solution) or train (for non-relativistic solution).

    As another fun, we can also show the ratio comparable to that 13-billions against one for the case I have presented lastly.

    In the ship reference frame, the load kinetic energy increases from zero to 0.5*50*(0.01c)<sup>2</sup>=0.0025c<sup>2</sup> J. In the rest reference frame (earth, in this case), this same load kinetic energy increases from 12.5c<sup>2</sup> to 12.878c<sup>2</sup>, an additional of 0.378c<sup>2</sup>. The ratio is 151 to 1 and there is no violation to any known physical laws, contrary to your earlier claim (Do you need to refresh your memory on that? Please do so in your discretion!).

    If you have a better calculations, this is just the right moment for you to present them. Show us your calculation supporting your claim that the kinetic energy change ratio of 13-billions to 1 or 151 to 1 doesn't make sense.
     
  17. Persol I am the great and mighty Zo. Registered Senior Member

    Messages:
    5,946
    We should call this MacM memory syndrome.
     
  18. MacM Registered Senior Member

    Messages:
    10,104
    Ditto.

    Nothing wrong with that, other than your innuendo that I have claimed it was illusion. I never did that or have you already forgotten your brief english lesson.? I realize that some people only learn by rote, you seem to be one of them.

    Only in your fairyland.

    Thanks for the laugh you imbelcile. The 13 B/1 ratio was the numbers given by James R., not my calculations at all. HeHeHe. Perhaps you should direct your comment to James R?

    Actually he will tell you what I am going to tell youo. Read, understand what you read and then speak. James R's, case was not the same as Petes case.

     
  19. Paul T Registered Senior Member

    Messages:
    460
    I can't believe you are this dumb. There is no problem with the 13 billions against 1 ratio, similarly there is no problem with 151 against 1 ratio for the case I presented recently. The problem is, you thought the ratio was a problem. This make you the dumbest of the dumb.

    Numerically, they are not the same, but they both contain similar large ratio of kinetic energy change.

    What a dumb question! In my calculation, I made reference to earth. One can do the same using Quasar (moving toward the spaceship at velocity, say, 0.6c) frame. The computation would give the same result with load velocity relative to the quasar increases at a slower rate compared to that relative to the spaceship frame. You can therefore consider that it is harder to accelerate relative to the quasar.

    Hope you have a little inteligence to grasp this as I am really not interested to waste my time to explain this to a dumb boy.
     
  20. MacM Registered Senior Member

    Messages:
    10,104
    Ditto. You continue to argue where there is no arguement. The ratio was never the issue. The ratio was James R's numbers, I just put them into perspective. Since you clearly are a rote learner, I'll politely repeat for the thrid time what the issue was.

    The issue was the changing of frames of referance and the impact on acceleration. If it required more energy to accelerate in a frame of referance where there is high relavistic velocity, then I was pointing out that your lack of knowing what is in motion in deep space could present a problem which shouold be detectable locally since (as in James R's case) your car does not have the 13B/1 times the horsepower to maintain acceleration when you just happen to turn in the direction of the Quasar.

    Think you have it now or will we need to go through it again?

    So what?

    That was my original statement. So why do you now pretend to be teaching me?

    The waste of time seems to be to get you to actually stay on point and to forget your meaningless attempts to ridicule or attack. You simply are ill equipped for the task.

    Perhaps you should just become a lurker and learn and shut the hell up.
     
  21. Paul T Registered Senior Member

    Messages:
    460
    Only you see that as a problem. Many (including myself) had explained to you that you have to consider relative to which reference frame your car or spaceship accelerates to.

    BTW, if you ever think that a constant power (or horsepower) will give your car a constant acceleration, I must also tell you that you are WRONG.

    What? Had you just changed your mind or you are simply suffering from chronic memory failure? Toward the beginning of this thread you said this:

    "What do you not understand about relative velocity causes an increase in energy required to accelerate. What do you not understasnd that Quasar ejecta moving at or > 0.95c is moving relative to us and that "According" to Relativity MUST affect our ability to accelerate. We see NO such affects. Not a peep."​
    You said that we see no such effect, implying that relativity must be wrong. What is your claim now, there is or there is no such effect?

    Many (again including myself) had explained to you that if your car acceleration is relative to that quasar (or quasar ejecta or whatever) then you will notice that it is harder to accelerate. You thought that by simply turn your car movement toward the incoming quasar you will notice the change of your acceleration rate. Of course not. The incoming quasar doesn't affect your car acceleration in some mystical way. To accelerate relative to that quasar means you must measure your car and quasar relative velocity change against the time. You have not yet done this (even in principle) and therefore your statement "We see NO such affects. Not a peep." is a foollish statement.
     
  22. MacM Registered Senior Member

    Messages:
    10,104
    Look idiot you nor nobody needed to explain to me the claims of Relativity. But they are just that "Claims" of Relativity. Stop talking like you know for fact things you clearly do not.

    1 - OK wise guy show me where I stated that constant horsepower produces constant acceleration. Stop talking to yourself and making up bullshit.

    2 - To suggest that constant horsepower does not produce constant accelertion is advocated only by Relativity and is supported only by data from particle accelerators.

    As I have pointed out many times relative velocity between a stationary driving energy source and the particle with a finite speed limit on the driving energy makes complete sense as to why the energy required would increase and velocity of the particle could never reach or exceed the speed of the driving energy. That fact has absolutely nothing to do with changing mass, etc however. It woudl be a decrease in energy transfer efficiency.

    That affect does not exist in a car nor a rocket. So when you drive your car near the speed of light and it requires more energy to maintain acceleration I will agree with you. Until you do you are advocating unsupported bullshit. So stop acting like you know what you do not.

    Interesting you sttart out saying I have changed my mind or lost my memory. You then end up asking what is my opinion. How about making up your mind.

    My opinion is you don't know and I don't know. I have already stated what Relativity claims and I have stated what my opinion is. Frankly my opinion is based on something other than unsupported dogma.

    Nobody needs to explain Relativity to me, especially you. Stop talking like you are a teacher. The problem is your assumption that the you can have two different accelerations simultaneously due to different frames of referance is based on the unsupported assumption that Relativity is valid and that that is what particle accelerators data dictates.

    Until you have demonstrated otherwise it is more logical to assume that forces and accelertion are unaffected by frame of referance. So neither view is proven. So stick it in your ear.
     
  23. Paul T Registered Senior Member

    Messages:
    460
    Hehe, it was just you who have mistakenly thought that you understand relativity. The fact is, you do not and given your attitude of "I know it better than anybody here", I would say you never will. And, I wasn't talking like I know for fact...I know the fact, sorry....hehehe.

    Why should I show you? I said: "If you ever think...." Oh boy, our english expert here seem to having problem with his english. Did I say you stated that constant horsepower produces constant acceleration? I didn't, mister retarded boy! As a matter of fact, I don't know...but I know you seem to think that constant horsepower produces constant acceleration and your are WRONG.

    Wrong again. This statement of yours not only shows your lack of understanding about relativity, but also general physics (yep, even newtonian physics).

    Let's do a simple calculation. Put a toy car (m = 0.5 kg) on flat floor. Apply a constant power of 1 watt to accelerate it.

    At t=0
    The toy car velocity v=0​
    At t=1s
    It gained 1 J of kinetic energy so that its velocity relative to the floor is 2m/s. Average acceleration from t=0 to t=1s is 2/1=2 m/s<sup>2</sup>​
    At t=2s
    It gained additional 1 J of kinetic energy. Its total kinetic energy is now 2 J (v=2.8284...m/s). Average acceleration from t=1 to t=2s is (2.8284-2)/1=0.8284... m/s<sup>2</sup>​
    As you can see from the above grade school level physics, constant power doesn't produce constant acceleration. So, your statement "constant horsepower does not produce constant accelertion is advocated only by Relativity" is FALSE. Even your grand children would be able to teach you that constant power does not produce constant acceleration.

    You are damn silly. You don't have to go through the "changing mass" routine to conclude that more energy has to be applied the faster the particle move in order to keep a constant acceleration.

    CRAP. This is your fairy land dream. Are you saying when particle move faster, it takes lesser energy (part of the energy just 'evaporate', doesn't increase the particle kinetic energy)? Even if all supplied energy is converted into particle kinetic energy, the faster the particle move the lesser would be its acceleration (given a constant power). And, this is the case even under newtonian mechanics.

    No, you will not agree with me. You are simply too dumb to even understand a very simple concept. More energy is required to give a car or rocket a constant acceleration. This is the case not only when the car or rocket move near to the speed of light.

    And now, what do you know if you don't even know that constant power doesn't produce constant acceleration?

    Your logic is worst than that of a cave man. Since you changing your mind so often how do I know you still think the same as you were weeks ago or even a few days ago? So, you should keep us posted about your current opinion and therefore I need to ask you again and again about it.

    You made your opinion based on your fairy land dream as I have EXPLAINED that to you above. Need more explanation?

    I agree, nobody needs to explain relativity to you. You don't even know enough basic physics and you want to know relativity? Learn the basic physics first then ask somebody to explain relativity to you.

    OMG. You don't even realize the root of the problem. Is there any acceleration if there is no reference frame? Can you accelerate your car relative to your car? I have given you enough EXPLANATION about this so you have to start thinking about it by yourself. Go and find it your own, then you will learn something. If I demontrate to you about every damn thing, you will remain as dumb as you are now.
     
    Last edited: Oct 26, 2004
Thread Status:
Not open for further replies.

Share This Page